Final Exam

Lakukan tugas rumah & ujian kamu dengan baik sekarang menggunakan Quizwiz!

In Graves' disease you would expect? (A) TSH to be decreased (B) Anti-TP to be increased (C) TSI to be decreased (D) T4 to be decreased

(A) TSH to be decreased

Sally is a 44-year-old who thinks she is beginning menopause because her menstrual periods have become irregular. Her major complaint is a lack of energy and weight gain. PE reveals dry skin, thinning hair, a puffy facial appearance and an enlarged, nontender thyroid. Her BP is 130/92, with a HR of 60. These findings are consistent with? (A) Grave's disease (B) Hypothyroidism (C) Plummer's disease (D) Thyrotoxicosis

(B) Hypothyroidism

Mrs. Waterman is a 64 year old with a history of varicosities and a single episode of thrombophlebitis. On inspection of both lower extremities you noted mottled pigmentation, erythema, varicosities, and ankle edema. What is your initial impression? (A) Cellulitis (B) Stasis dermatitis (C) Atopic dermatitis (D) Contact dermatitis

(B) Stasis dermatitis

The most serious side effects of both PTU and methimazole is: (A) Skin rash (B) Diarrhea (C) Agranulocytosis (D) Hepatitis

(C) Agranulocytosis

Grave's disease is caused by: (A) Viral infection (B) Use of lithium (C) An autoimmune response (particularly the production of thyroid-stimulating immunoglobulins that bind to the TSH receptor and mimic the effects of TSH) (D) Excessive ingestion of thyroid hormone

(C) An autoimmune response (particularly the production of thyroid-stimulating immunoglobulins that bind to the TSH receptor and mimic the effects of TSH)

Mrs. H is diagnosed with hypothyroidism and is placed on levothyroxine 0.05 mg per day. After a week she calls you and tells you she hasn't seen any improvement and wants to discontinue her medication. Your best response would be to: (A) Add propranolol to her regimen (B) Change to desiccated thyroid (C) Increase her dosage to 0.125 mg/day (D) Encourage her to take this dose for at least another week.

(D) Encourage her to take this dose for at least another week.

Upon initial diagnosis of Grave's Disease which medication should be ordered immediately for symptom control? (A) PTU (B) Cytomel (C) 131 I (D) Propranolol

(D) Propranolol

A 60-year-old man comes to the clinic to establish care. He has had grittiness, stinging, and redness in both eyes for several years. He has no history of eye surgery or injury, has not taken any medications or ocular medicaments, and has not changed eyewear. He states that the eye symptoms are not seasonal and they often accompany a facial rash. On examination, he has bilateral blepharitis and minimal conjunctival injection. There are erythematous patches on his cheeks and nose with scattered telangiectasias. There are papules and pustules but no comedones within these areas. The nose is slightly bulbous. Vision is 20/40 in both eyes with corrective glasses (his baseline). Which one of the following is the most likely diagnosis that explains his ocular symptoms? (A) Acne vulgaris (B) Impetigo (C) Seborrheic dermatitis (D) Rosacea

(D) Rosacea

The best guide to adequacy of the dosage of antithyroid medication in thyrotoxicosis is: (A) Thyroid antibodies (B) T3 (RIA) (C) TSH (D) T4

(D) T4

Following the initial questions that you ask Mrs. Street, Lauren urges her mother to talk with you "about a specific concern," but Mrs. Street appears reluctant. How would you respond to the daughter's request? Choose the single best answer. A. Ask Mrs. Street if it is OK if the daughter talks to you about her concerns B. Ask the daughter to step out of the room with you to another room so that you can confer out of the hearing of the patient C. Bring the nurse in the room and ask her to explain the situation further D. Excuse yourself and ask Dr. Jones (your preceptor in clinic) what would be the best way to conduct the visit

A. Ask Mrs. Street if it is OK if the daughter talks to you about her concerns

"Given her long-standing urinary symptoms as well as her progressive worsening, what type(s) of urinary incontinence does Mrs. Street have?" Select all that apply. A. Functional UI B. Mixed UI C. Overflow/Bladder Outlet Obstruction UI D. Stress UI E. Urgency Predominant UI

A. Functional UI & E. Urgency Predominant UI

A patient presents with CVA tenderness and a several-day history of high fever, chills, and dysuria. Which of the following diagnoses is most likely given the above information? A. Pyelonephritis B. Cystitis C. Renal calculi D. Bladder tumor

A. Pyelonephritis

The use of a thiazolidinedione is not recommended in all of the following except: 57M taking a nitrate 62F with heart failure 45M on insulin 35 yo pt with newly dx'ed DM2

35 yo pt with newly dx'ed DM2

Osteoporosis is defined as having a bone density more than ___ standard deviation(s) below the average bone mass for a healthy young adult. 1 1.5 2.5 4

2.5

A 78 y/o old patient comes to the clinic because he fell yesterday. He is worried he will fall again since he is not sure why he fell in the first place. Prior to his fall, he reports feeling weak and dizzy. Medical history is significant for a-fib, diabetes, HTN CAD, CKD, and spinal stenosis. Medications are acetaminophen as needed, apixaban, carvedilol, insulin glargine and nifedipine. Physical exam shows a well-groomed man in NAD. His hearing is intact with bilateral hearing aids, vision is 20/20 with corrective lenses and dentition. He has normal heart and lung exams. He has a normal gait without using any assisted devices. Get-up and go test is normal with no use of arms to help him stand. He is alert and orientedx3 and has no focal neuro deficits. Results of lab tests include EKG-normal, CBC-normal, sodium 136, Cr 1.7, eGFR 40, glucose 70 ad hBA1c 6.5% What is the next best step to present another fall in this man? 1. change A1C target to 7% 2. Follow up with an audiologist 3. Recommend use of a walker 4. Referral for PT 5. Start meclizine for his dizziness

1. change A1C target to 7%

What is the recommended daily calcium intake for women >50y.o.? 800mg 1000mg 1200mg 1500mg

1200mg

Among the following, who is at the greatest risk of developing seborrheic dermatitis? 15M in a rural setting 34F who smokes 2 packs per day 48M truck driver 72M with Parkinson's

72M with Parkinson's

hgbA1c best provides info no glucose control over the past: (it didn't say over what time period? Days?) 21 to 47 48 to 68 69 to 89 90 to 120

90 to 120

A lumbosacral XR would be most helpful to aid in the dx of new-onset LBP in which of the following? A 49M 1 day after performing rigorous yardwork A 52F who slipped on an icy sidewalk A 54M who is obese who reports pain radiating to the butt A 64F reporting pain after attempting to lift her grandson

A 52F who slipped on an icy sidewalk

A 46-year-old male presents to your office. He was sent by an orthopedist. He has recently had surgical fixation of a humerus fracture. The patient has been going to physical therapy and has been developing a rash on his arm after therapy that disappears shortly after returning home. He does not have the rash prior to therapy. The patient denies fevers and chills, and his incision is well healed, with no signs of infection. Of note, the patient has been experiencing more hand edema than the average patient and has had edema wraps used at the end of therapy to help with his swelling. The wraps are made of a synthetic plastic material. The rash the patient gets is erythematous and blotchy, not raised; it is on the operative upper extremity. What is the most likely diagnosis? A ] Contact dermatitis. B ] Atopic dermatitis. C] Seborrheic dermatitis. D] Psoriasis.

A ] Contact dermatitis.

The mechanism of action of the DPP-4 inhibitors is as: A drug that increases levels of incretin, increasing synthesis and release of insulin from pancreatic beta cells A product virtually identical in action to sulfonylureas A drug that increases insulin action in the peripheral tissues and reduces hepatic glucose production A facilitator of renal glucose excretion

A drug that increases levels of incretin, increasing synthesis and release of insulin from pancreatic beta cells

The mechanism of action of the GLP-1 agonist is as: A drug that stimulates insulin production in response to an increase in plasma glucose A product virtually identical in action to sulfonylureas A drug that increases insulin action in the peripheral tissues and reduces hepatic glucose production A facilitator of renal glucose excretion

A drug that stimulates insulin production in response to an increase in plasma glucose

A 37yo afebrile woman w/no health problems p/w dysuria and frequency. Her UA findings include results positive for nitrites and leukocyte esterase. You evaluate these results and consider that she likely has: Purulent vulvovaginitis A gram-negative UTI Cystitis caused by staphylococcus saprophyticus Urethral syndrome

A gram-negative UTI

The zoster vaccine (zostavax) is: In inactivated/killed virus vaccine A conjugate vaccine containing a virus-like particle (VLP) A live, attenuated vaccine An inactivated toxin vaccine

A live, attenuated vaccine

The mechanism of action of sulfonylureas is as: An antagonist of insulin receptor site activity A product that enhances insulin release A facilitator of renal glucose excretion An agent that can reduce hepatic glucose production

A product that enhances insulin release

Which of the following is the most accurate info in caring for a 40yo man with cystitis? This is a common condition in men of this age A gram-positive organism is the likely causative pathogen A urological evaluation should be considered Pyuria is rarely found

A urological evaluation should be considered

Generally, testing for type 2 DM in asymptomatic undiagnosed people older than 45 years should be conducted every ___. Year 3 years 5 years 10 years

3 years

A 44yo woman p/w pyelonephritis. The report of her UA is least likely to include: WBC casts Positive nitrites 3+ protein Rare RBCs

3+ protein

Approximately what % of pts with radiological findings of OA of the knee will report having symptoms? 25% 50 70 95

50

In a healthy person, what percentage of the body's total daily physiological insulin secretion is released as basally? 10-20% 25-35 50-60 75-85

50-60

Which of the following pts has prediabetes? 70M with fasting glucose of 99 84F with a 1hr postprandial glucose of 98 33M with a hgbA1c of 5.4% 58F with a 2hr postprandial glucose of 152

58F with a 2hr postprandial glucose of 152

Consideration should be given to setting a1c goal in 22M w/ 8y h/o DM1 with no comorbidities equal to or less than: 5.5% 6 6.5 7

6

At minimum, at what interval should TSH be reassessed after a levothyroxine dosage is adjusted? 1-2 weeks 2-4 weeks 4-6 weeks 6-8 weeks

6-8 weeks

5. What is the most effective antifungal agent used in the treatment of tinea capitis? A. Terbinafine orally (oral is standard for tinea capitis) B. Clotrimazole 1% cream C. Triamcinolone 0.0.25% cream D. Nystatin cream

A. Terbinafine orally (oral is standard for tinea capitis)

Which of the following statements is true regarding the treatment of carpal tunnel syndrome? A. The goal of treatment is to prevent flexion and extension movements of the wrist. B. Splints are not used in carpal tunnel syndrome, because they restrict complete movement of the fingers and wrist. C. Corticosteroid injections are encouraged in the treatment of carpal tunnel syndrome.

A. The goal of treatment is to prevent flexion and extension movements of the wrist.

A patient is diagnosed with overactive bladder. Which of the following instructions should be given to this woman? A. "Limit the amount of water that you drink." B. "Eliminate caffeine from your diet." C. "Wear panty liners." D. "You should not attempt pregnancy."

B. "Eliminate caffeine from your diet."

The clinician has been doing diabetic teaching for a patient with type 1 diabetes with no retinopathy. Which statement by the patient would indicate that teaching has been effective? A. "As long as I don't need glasses, I don't have to worry about going blind." B. "I know I need to have my eyes checked every 2 years." C. "My primary doctor will check my eyes." D. "I will see my eye doctor when my vision gets blurry."

B. "I know I need to have my eyes checked every 2 years."

Which of the following diagnostic tests should be ordered for a patient suspected of having bladder cancer? A. Kidneys, ureter, bladder x-ray B. Cystoscopy with biopsy C. Magnetic resonance imaging D. Urine tumor marker (NMP22)

B. Cystoscopy with biopsy

1. An 85-year-old patient comes to the clinic because of a three-month history of increased urinary leakage. She reports that she has had chronic urinary incontinence for several years, which she manages supportively with daily incontinence products. Medical history is additionally significant for hypertension (HTN), congestive heart failure (CHF), and atrial fibrillation. Medications are apixaban, senna, lisinopril, and furosemide. Her cardiologist started furosemide twice daily for decompensated congestive heart failure four months ago. Vital signs are normal. Physical examination shows bilateral mild pedal edema. Her genitourinary examination showed mild vaginal atrophy. The examination was otherwise normal. Results of laboratory/diagnostic studies show normal urinalysis and post-void residual on bladder scan. In addition to pelvic floor physical therapy, what would be the next best step to help treat her symptoms? Choose the single best answer. A. Decrease water intake B. Discuss medications with cardiologist C. Increase her incontinence products use D. Recommend cystoscopy E. Start oxybutynin

B. Discuss medications with cardiologist

Her post-void residual volume is: 150 ml after a 150 ml voided volume. Question What is the result of Mrs. Street's post-void residual volume? Choose the single best answer. A. Elevated and an in-dwelling catheter is warranted B. Elevated, but may be normal for her age C. Needs to be repeated, urinated volume is too low D. Normal E. None of the above

B. Elevated, but may be normal for her age

An 86 y/o patient presents to the ED after her family found her down in her home. She lives alone, her fall was unwitnessed. Prior to this, she was able to do her usual activities. The last time anyone had spoken to her was 36 hours before. She has a history of DMII, and CKD stage 2. Over the past 15 years, she has taken glimepiride, HCTZ, metoprolol, aspirin, and rosuvastatin. Her family has recently been worried that her memory is worsening. They don't think she checks her blood sugars regularly. Her pulse is 107. BP is 106/54. Weight is 45.3kg. Height is 162c. Her BMI is 17.3. Other VS WNL. Physical exam shows a thin, lethargic female. There is temporal wasting and proximal muscle bulk is decreased. Mucous membranes are dry. Cardiovascular exam shows sinus tachycardia. Lungs are clear to auscultation. Abdominal exam is benign. There is no edema. Skin is dry. Today labs studies show serum glucose of 40, potassium of 5.0, BUN 30, Cr 1.5, Creatine kinase is mildly elevated at 200. Which of her chronic medications is most likely responsible for this event? A. Aspirin B. Glimepiride C. HCTZ D. Metoprolol E. Rosuvastatin

B. Glimepiride

A 37 y/o male HS teacher complains of an acute onset of pain and redness of his L wrist. He denies trauma to the wrist. He reports that the condition occurred on the same wrist before. The symptoms started that morning after he went to a birthday party the previous evening. He reports taht he drank a few glasses of white wine. During the physical exam, the L wrist is red, swollen and tender on palpation. The skin is intact but feels warm to touch. What is most likely the diagnosis? A. Wrist sprain B. Gout C. Osteoarthritis D. Septic joint

B. Gout

A 20-year-old female presents to your urgent care clinic complaining of a cat bite. The patient recently adopted a cat. She was playing with the cat yesterday when the cat bit her on the arm. What antibiotic should be prescribed to prevent infection? A] Augmentin. B ] Amoxicillin. C] Bactrim. D ] No antibiotic is necessary.

A] Augmentin.

Which of the following is not a recommended option to make cosmetic improvements or phymatous rosacea? Laser peel Ablative laser surgery Surgical shave technique Mechanical dermabrasion

Ablative laser surgery

The Tinel test is best described as: Reproduction of symptoms with forced flexion of the wrists Abnormal tingling when the median nerve is tapped Pain on internal rotation Palmar atrophy

Abnormal tingling when the median nerve is tapped

Psoriasis vulgaris is a chronic skin disease caused by: Bacterial colonization Absence of melanin Accelerated mitosis Type I HSR

Accelerated mitosis

Tamsulosin (Flomax) is helpful in the TX of BPH because of its effect on: Bladder contractility Prostate size Activity at select bladder receptor sites Bladder pressure

Activity at select bladder receptor sites

Which of the following best describes ethnicity and insulin sensitivity? Little variation exists in insulin sensitivity among different ethnic groups African americans are typically less sensitive to the effects of insulin compared to europeans Mexican americans are likely the most insulin-sensitive ethnic group in north america The degree of insulin sensitivity has little influence on insulin production

African americans are typically less sensitive to the effects of insulin compared to europeans

An obese Asian patient with a BMI of 33 complains of fatigue and excessive thirst and hunger. You suspect type 2 diabetes mellitus. Initial testing to confirm diagnosis can include: Fasting plasma glucose level Glycated hemoglobin level (A1C) Oral glucose tolerance testing All of the above

All of the above

You perform a DRE on a 72yo and find a lesion suspicious for prostate cancer. The findings are described as: A rubbery, enlarged prostatic lobe An area of prostatic induration A boggy gland Prostatic tenderness

An area of prostatic induration

What is the treatment for functional incontinence?

An assistant who can recognize voiding cues

In counseling a postmenopausal woman, you advise her that systemic estrogen therapy users can possibly experience: An increase in breast cancer rates with long-term use Reduction in HDL A 10% increase in bone mass No change in the occurrence of osteoporosis

An increase in breast cancer rates with long-term use

TSH is released by the: Thyroid follicles Adrenal cortex Hypothalamus Anterior lobe of the pituitary

Anterior lobe of the pituitary

The most important aspect of skin care for individuals with atopic dermatitis is: Frequent bathing with antibacterial soap Consistent use of medium-potency to high-potency topical steroids Application of lubricants Treatment of dermatophytes

Application of lubricants

In counseling a patient with atopic dermatitis, you suggest all of the following can be used to alleviate symptoms of a flare except: The use of oral antihistamines Applying a heat pad on the affected region for 30 minutes The use of topical corticosteroids Applying cool, wet dressings made from a clean cloth and water the affected area

Applying a heat pad on the affected region for 30 minutes

Patients with urethral stricture often report urine loss: With exercise During the day Associated with urgency As dribbling after voiding

As dribbling after voiding

Patients with urge incontinence often report urine loss: With exercise At night Associated with a strong sensation of needing to void As dribbling after voiding

Associated with a strong sensation of needing to void

Risk factors for becoming a perpetrator of elder maltreatment include all but: A high level of hostility about the caregiver role Poor coping skills Assumption of caregiving responsibilities at a later stage of life Maltreatment as a child

Assumption of caregiving responsibilities at a later stage of life

hgbA1c should be tested: At least annually for all patients At least 2x/year in pts who are meeting tx goals and who have stable glycemic control Monthly in pts whose tx has changes or who are not meeting glycemic goals Only via standardized lab testing because of inaccuracies a/w point-of-service testing

At least 2x/year in pts who are meeting tx goals and who have stable glycemic control

A 55 y/o male with HTN and DM II presents to the office for routine f/u. He has a history of bladder cancer that was treated by resection and chemotherapy five years ago. He is currently asymptomatic. He takes lisinopril, aspirin and metformin. His HR is 76, BP 128/78, BMI 29. The rest of his physical exam is unremarkable. Labs show: glucose 142, UA negative for leukocyte esterase, nitrite, bili blood and trace protein Microalbumin 275 mcg/g Lipid profile: TC 295, LDL 35, LDL 220, Tri 200 HbA1c 7.2%^ Which of the following is the most appropriate medication to initiate in this patient? Amlodipine Atorvastatin Losartan Niacin Pioglitazone

Atorvastatin

Hypothyroidism most often develops as a result of: Primary pituitary failure Thyroid neoplasia Autoimmune thyroiditis Radioactive iodine exposure

Autoimmune thyroiditis

The findings of a painless thyroid mass and TSH level of < 0.1IU/mL in a 35F is most consistent with: Autonomously functioning adenoma Graves' disease Hashimoto's disease Thyroid malignancy

Autonomously functioning adenoma

The best screening test for both hyperthyroidism and hypothyroidism is: A. Free T4 (thyroxine) B. Thyroid-stimulating hormone C. Thyroid profile D. Palpation of the thyroid gland

B. Thyroid-stimulating hormone

A 27-year-old female presents with a chief complaint of burning and pain on urination. She has no previous history of urinary tract infection (UTI). What are some additional symptoms consistent with a diagnosis of lower UTI? A. Thick, green discharge B. Fever, chills, costovertebral angle (CVA) tenderness C. Blood in urine and frequency D. Foul-smelling discharge, perineal itch

C. Blood in urine and frequency

Which of the following tests is considered the gold standard for definitively diagnosing osteoporosis? A. Bone alkaline phosphatase levels B. Urinary N-telopeptide assay C. Bone mass density measurement by densitometry D. Magnetic resonance imaging

C. Bone mass density measurement by densitometry

Among patient's with nephrolithiasis, what the most common type of kidney stone? A. Struvite B. Cystine C. Calcium Oxalate D. Uric Acid

C. Calcium Oxalate

One of the initial steps in assessing patients with musculoskeletal complaints is to determine whether the complaint is articular or nonarticular in origin. Which of the following is an example of an articular srtucture? A. Fascia B. Bone C. Synovium D. Tendons

C. Synovium

All of the following increase the risk of UTI in women, except: A. Frequent sexual intercourse B. Pregnancy C. Vaginal douching D. Spermicide use w/ diaphragms

C. Vaginal douching

Are the following findings in hypothyroidism/ hyper or both? Goiter, atypical presentation in an elderly person, change in mental status

BOTH

Mood or mentation change Is this a finding in (1) hyperthyroidism, (2) hypothyroidism or (3) both?

BOTH

Which of the following should be the goal measurement in treating a 45M with DM and HTN? BP <140/90 hgbA1c = or > 7% trigs 200-300 HDL 35-40

BP <140/90

Tom, age 50, is complaining of an itchy rash that occurred about a half hour after putting on his leather jacket. He recalls having a slightly similar rash last year when he wore his jacket. The annular lesions are on his neck and both arms. They are erythematous, sharply circumscribed, and both flat and elevated. His voice seems a little raspy, although he states that his breathing is normal. What is your first action? A ] Order a short course of systemic corticosteroids. B] Determine the need for 0.5 mL 1:1000 epinephrine subcutaneously. C ] Start daily antihistamines. D ] Tell Tom to get rid of his leather jacket.

B] Determine the need for 0.5 mL 1:1000 epinephrine subcutaneously

You see a 54M c/o LBP and who is dx'ed with acute lumbosacral strain. Which of the following is the best advise to give about exercising? You should not exercise until you are free of pain Back-strengthening exercises can cause mild muscle soreness Electric-like pain in response to exercise is to be expected Conditioning exercises should be started immediately

Back-strengthening exercises can cause mild muscle soreness

Nondairy sources of calcium include all of the following except: Tofu Spinach Brown rice Sardines

Brown rice

You are seeing a 55 y/o F in the office today for her annual physical exam. Her medical history includes HTN, DMII, hyperlipidemia and obesity. As you review her chart, you note that she is up to date on her diabetic foot exam, eye exam for retinopathy, colonoscopy and mammogram. What additional screening, according to the USPSTF should be performed for all adults? Breast cancer Depression Hepatitis C virus Human immunodeficiency virus Osteoporosis

Depression

The mechanism of action of radioactive iodine in the tx of Graves' disease is to: Destroy the overactive thyroid tissue Reduce production of TSH Alter thyroid metabolic rate Relieve distress caused by increased thyroid size

Destroy the overactive thyroid tissue

Factors that contribute to urge incontinence include: Detrusor overactivity Pelvic floor weakness Urethral stricture UTI

Detrusor overactivity

You are seeing a male in the office for a follow-up for depression. He has been participating in psychotherapy and taking 20 mg of citalopram daily for the last four weeks. He notes some improvement in his depressive symptoms, including increased energy levels, improved interest in activities, improved sleep, and improved appetite. While he has been pleased with the progress he has made with medications and psychotherapy, he is bothered by a recent decrease in libido and ability to achieve orgasm. He thinks that this may be related to citalopram, as these new symptoms began one week after starting this medication for depression. Which of the following is the most-appropriate step in medication management? Continue current dose of citalopram Discontinue citalopram and continue with only psychotherapy Discontinue citalopram and start bupropion Discontinue citalopram and start fluoxetine Increase dose of citalopram from 20 mg/day to 40 mg/day

Discontinue citalopram and start bupropion

The leading cause of death among people with RA is: Infection CV events Cancer Renal failure

CV events

In children and elderly, which of the following conditions can contribute to bladder instability and increase the risk of a UTI? Constipation URI Chronic diarrhea Efficient bladder emptying

Constipation

You have initiated therapy for an 18M w/acne and have prescribed doxycycline. He returns in 3 weeks c/o no improvement. Your next action is to: Counsel him that 6-8 weeks of treatment is often needed before significant improvement Discontinue doxycycline and initiate minocycline therapy Advise him that antibiotics are likely not an effective treatment for him and should not be continued Add a second antimicrobial agent such as trimethoprim-sulfamethoxazole

Counsel him that 6-8 weeks of treatment is often needed before significant improvement

An 86-year-old woman is seen in the clinic for recurrent hematuria. The provider suspects bladder cancer. Which of the following data from the history is considered a risk factor for this type of cancer? A. History of alcoholism B. Sedentary lifestyle C. Obesity D. 65-year smoking history

D. 65-year smoking history

One of the most frequent presenting signs/symptoms of osteoporosis is: A. Goiter B. Abnormal serum calcium C. Elevated urine biochemical markers D. Bony fracture

D. Bony fracture

The clinician is caring for Diane, a 22-year-old woman who presents with an injured ankle. Diane asks the clinician if she will need an x-ray. The clinician explains to Diane that an x-ray is not always necessary for an injured ankle and that the decision to obtain radiographs is dependent on the examination and Diane's description of her injury. Which of the following clues in Diane's examination or history would alert the clinician to the need for obtaining radiographs? A. Ability to bear weight immediately after the injury B. Development of minor ankle swelling after the injury C. Slight bruising over the injury site 2 days after injury D. Crepitation with palpation or movement of the ankle

D. Crepitation with palpation or movement of the ankle

A 55-year-old male with a history of hypertension, type II diabetes mellitus, and coronary artery disease presents to your preceptor's office with a new diagnosis of depression. He is divorced, but his adult daughter is in rehab for treatment of a substance use disorder, and he has full custody of her two young children. He previously enjoyed fishing and target shooting, but has not had interest in these activities lately. In addition to assessing him for suicidal ideation, you assess factors that increase the risk of suicide completion and for factors that are protective. Which factors are considered protective against his completion of suicide? a.Access to handguns b. Chronic medical conditions c. Divorced marital status d. Feelings of responsibility towards family e. Male sex

D. Feelings of responsibility towards family

Dr. Jones asks you, "What is the most important finding from the urinalysis that may be contributing to Mrs. Street's symptoms?" Choose the single best answer. A. Specific gravity B. Urine blood C. Urine color D. Urine glucose E. Urine leukocyte esterase

D. Urine glucose

Which of the following is the best factor to consider in assessing skin lesions? A. location of the lesion B. type of lesion C. pattern of the lesion D. all of the above

D. all of the above

One of the most common trigger agents for contact dermatitis is: Exposure to nickel Use of fabric softener Bathing with liquid body wash Eating spicy foods

Exposure to nickel

You consider prescribing insulin glargine (toujeo, lantus) because of its: Extended duration of action Rapid onset of action Ability to prevent diabetic end-organ damage Ability to preserve pancreatic function

Extended duration of action

Physical exam findings in patients with Graves' disease include: Muscule tenderness Coarse, dry skin Eyelid retraction Delayed relaxation phase of the patellar reflex

Eyelid retraction

You see an obese 25M with acanthosis nigricans and consider ordering: FBS LFT RPR ESR

FBS

Acanthosis nigricans is commonly noted in all of the following areas except: Groin folds Axilla Nape of the neck Face

Face

You are seeing a 17F and consider her risk factors for type 2 DM would include all of the following except: Obesity Native American ancestry Family history of type 1 DM Personal history of polycystic ovary syndrome

Family history of type 1 DM

Analgesia options for a pt with shingles can include all of the following except: Topical lidocaine gel 5% with oral Tylenol Burow's solution with high-potency oral NSAID Burow's solution with an oral opioid Fentanyl transdermal patch and a topical medium-potency corticosteroid on the affected area

Fentanyl transdermal patch and a topical medium-potency corticosteroid on the affected area

Which of the following is the most cost-effective method of distinguishing a malignant from a benign thyroid nodule? Ultrasound MRI Fine-needle aspiration biopsy Radioactive iodine scan

Fine-needle aspiration biopsy

A common site for atopic dermatitis in an adult is on the: Dorsum of the hand Face Neck Flexor surfaces

Flexor surfaces

An 85 y/o Male patient comes to the clinic for a wellness visit. Medical history is significant for HTN, DMII, and peripheral neuropathy. Medications are lisinopril, metformin, gabapentin, amlodipine and levothyroxine. VS are normal including a BP 128/76. Weight is 52kg. Height is 150cm. BMI 23.1. Physical exam is unremarkable. Results of labs show Cr 1.3, BUN 18, glucose 172 and the rest of the BMP is WNL. HbA1C 7.5%. Using the CKD-EPI equation, calculate the patients eGFR. Use an online calculator such as the one here https://www.mdcalc.com/calc/3939/ckd-epi-equations-glomerular-filtration-rate-gfr. Which of the patient's medications need to be adjusted because of the patient's renal function? Amlodipine Gabapentin Levothyroxine Lisinopril Metformin

Gabapentin

A 24M was bit in the thigh by a raccoon in the woods. Exam reveals a 1cm deep wound and is oozing bright red blood. Your next best action is: Administer high-dose parenteral penicillin Initiate antibacterial prophylaxis with amoxicillin Give rabies immune globulin and rabies vaccine Suture the wound after proper cleansing

Give rabies immune globulin and rabies vaccine

Which of the following meds should be used with caution in a person with severe sulfa allergy? Metformin Glyburide Rosiglitazone NPH insulin

Glyburide

An adolescent athlete reports a sudden onset of posterior R thigh pain. The injury occurred while she was running during a soccer game. She denies falling. She is limping and walking slowly. On physical exam, the midportion of the R hamstring is tender to palpation with some bruising noted. Which of the following conditions is most likely in this patient? Hamstring muscle strain Baker's cyst Quadriceps muscle strain Piriformis syndrome

Hamstring muscle strain

Compared with UTI in younger women, uncomplicated UTI in an elderly woman is more likely to be a/w each of the following except: New-onset urinary incontinence Delirium Weakness Hematuria

Hematuria

Which organ system is not affected by chronic NSAID use? Cardiovascular system Renal system Immune system GI system

Immune system

Interventions in microalbuminuria for a person with DM includes which of the following? (more than 1 can apply) Improved glycemia control Strict dyslipidemia control Use of an optimized dose of an ACEI or ARB Use of an ACEI with an ARB

Improved glycemia control Strict dyslipidemia control Use of an optimized dose of an ACEI or ARB

A 36M has 2 furuncles x3 days that are increasing. No systemic symptoms. You advise: Incision and drainage Systemic antibiotic empirically Topical antibiotic Aspiration of lesion contents and rx of a systemic abx based on culture results

Incision and drainage

Mr. Samuels is a 58M with DM2 who is using a single 10 unit daily dose of the long-acting insulin glargine. His fasting glucose has been 141-180. Which of the following best describes the next step in his therapy? Continue on the current glargine dose Increase his glargine dose by 4 units per day Increase his glargine dose by 1 units per day Increase his glargine dose by 6 units per day

Increase his glargine dose by 4 units per day

All are common signs of osteoporosis except: Gradual loss of height with stooped posture Hip or wrist fracture Increase in waist circumference Pt report of back pain

Increase in waist circumference

Physical exam consistent with atopic dermatitis is a diagnosis that: Requires a skin culture to confirm contributing bacterial organisms Should be supported by a biopsy of the affected area Necessitates obtaining peripheral blood eosinophil level Is usually made by clinical assessment alone

Is usually made by clinical assessment alone

Which of the following statements is most accurate concerning RA? Joint erosions are often evident on radiographs or MRI RA is seldom a/w other autoimmune diseases A butterfly-shaped facial rash is common Parvovirus B19 infection can contribute to its development

Joint erosions are often evident on radiographs or MRI

With the straight-leg-raising test, the NP is evaluating tension on which of the following nerve roots? L1 and L2 L3 and L4 L5 and S1 S2 and S3

L5 and S1

Which of the following do you expect to find in the assessment of the person with urticaria? Eosinophilia Low ESR Elevated TSH Leukopenia

Eosinophilia

Oral antimicrobial tx recommended for rosacea include all of the following except: Metronidazole Levofloxacin Erythromycin Doxycycline

Levofloxacin

You see a 34M with atopic dermatitis localized primarily on the arms who c/o severe itching that is worse at night and interferes with sleep. You recommend: Taking a bedtime dose of antihistamine Takin a bedtime dose of acetaminophen Taking a hot shower prior to bedtime Applying a warm compress to the affect areas 30 mins prior to bedtime.

Taking a bedtime dose of antihistamine

A patient with type 1 DM is on regular insulin and NPH insulin (not premixed but separate) injected twice a day. The first dose is injected before breakfast, and the second dose is injected at bedtime. The blood sugar results from the patient's diary (fasting, before lunch, dinner and bedtime) show that the lunchtime values are higher than normal. Which insulin dose should be increased or decreased?

The NPH component of the morning dose should be increased. Why? Regular insulin peaks b/t breakfast and lunch (most of it is gone by lunchtime). In contrast, NPH insulin peaks between 6 and 14 hours. Therefore, it will cover the postprandial spike after lunch.

In prescribing levothyroxine therapy for an elderly patient, which of the following statements is true? Elderly persons require a rapid initiation of levothyroxine therapy TSH should be checked about 2 days after dosage adjustment The levothyroxine dose needed by elderly persons is 75% of less of that needed by younger adults TSH should be suppressed to a nondetectable level

The levothyroxine dose needed by elderly persons is 75% of less of that needed by younger adults

A 48F w/newly dx'ed hypothyroidism asks about a "natural thyroid" med she read about called desiccated thyroid. As you counsel her about this med, you consider all of the following except: This product contains a fixed dose of T3 and T4 The med is a plant-based product Its pharmacokinetics differ significantly when compared to levothyroxine The majority of the study on treatment for hypothyroidism has been done using levothyroxine

The med is a plant-based product

Concerning BPH, which of the following statements is true? a. DRE is accurate in diagnosing the condition b. The use of a validated patient symptom tool is an important part of diagnosing the condition c. Prostate size directly correlates with symptoms and bladder emptying d. Bladder distension is usually present in early disease

The use of a validated patient symptom tool is an important part of diagnosing the condition

Pertaining to the use of sliding-scale insulin in response to elevated blood glucose, which of the following best describes current best practice? The use of this type of sliding-scale insulin therapy is discouraged as this method treats hyperglycemia after it has already occurred Sliding-scale insulin in response to elevated glucose is a safe and helpful method of treating hyperglycemia Delivering insulin in this manner is acceptable within the acute care hospital setting only The use of the ISS is appropriate in the tx of DM1 only

The use of this type of sliding-scale insulin therapy is discouraged as this method treats hyperglycemia after it has already occurred

You advise a 58F with hypothyroidism about the correct use of levothyroxine. She has takes a calcium supplement. All of the following should be shared with the patient except which instruction? Take the med on an empty stomach To help with adherence, take your calcium supplement at the same time as your thyroid med. You should take the med at approximately the same time every day Do not take your med with soy milk

To help with adherence, take your calcium supplement at the same time as your thyroid med.

Pharmacologic intervention for patients with urge incontinence includes: Tamsulosin (Flomax) Tolterodine (Detrol) Finasteride (proscar) Pseudoephedrine

Tolterodine (Detrol)

Regarding the previous case, all of the following can be used to treat his condition, except: Oral glucocorticoid Topical glucocorticoid NSAIDs Colchicine

Topical glucocorticoid

A 62 y/o male presents to this PCP for a follow up of his DMII. He is asymptomatic and is currently taking metformin. His pulse is 76, BP 142/92 and BMI 29. The remainder of shi physical exam is WNL. Lab results are: Glucose 132, UA negative for leukocyte esterase, negative nitrate, negative bili, negative blood, trace protein, microalbumin 275 mcg/mg. Initiation of which drug from which of the following classes would be most appropriate for this patient? a. Angiotensin converting enzyme inhibitor b. Beta blocker c. Calcium channel blocker d. Thiazide diuretic e. Vasodilator

a. Angiotensin converting enzyme inhibitor

A 68 year old patient comes to the clinic because of back pain for the past week that has made it difficult to walk. He describes his back pain as intermittent and worse when getting dressed, especially tying his shoes. He reports no falls, fevers, dysuria or hematuria. He reports helping his son move 8 days ago. Medical history for BPH, HTN, hypothyroidism, MM (in remission) and osteoarthritis. Medications are acetaminophen as needed, ibuprofen as needed, Synthroid, lisinopril and tamsulosin. Vital signs are normal. Physical exam shows a man holding his lower back and grimacing with movements. No CVA tenderness. Strength and sensation are intact and similar over bilateral paraspinal lumbar region. Gait is slow but normal. What is the most likely diagnosis for this patient? a. Lumbar muscle strain b. Multiple myeloma c. Nephrolithiasis d. Prostatitis e. Sciatic

a. Lumbar muscle strain

Which form of urinary incontinence is most common in elderly persons? Stress Urge Iatrogenic Overflow

Urge

Leakage with the urge or the feeling that you needed to empty your bladder, but could not get to the toilet fast enough.

Urge only or urgency predominant

Which of the following is most likely to be part of the clinical presentation of an otherwise healthy 27yo woman w/uncomplicated lower UTI? Urinary frequency Fever Suprapubic tenderness Lower GI upset

Urinary frequency

When prescribing tretinoin (Retin-A), the NP advises the patient to: Use it with benzoyl peroxide to minimize irritating effects Use a sunscreen because the drug is photosensitizing Add a sulfa-based cream to enhance anti-acne effects Expect a significant improvement in acne lesions after approximately 1 week of use

Use a sunscreen because the drug is photosensitizing

Common physical findings of SLE include all but: Weight gain Joint pain and swelling Fatigue Facial rash

Weight gain

You examine a pt with psoriasis vulgaris and expect to find the following lesions: Lichenified areas in flexor areas Well-demarcated plaques on the knees Greasy lesions throughout the scalp Vesicular lesions over the upper thorax

Well-demarcated plaques on the knees

An urticarial lesion is usually described as a: Wheal Plaque Patch Papule

Wheal

Changes to the joint during OA can typically include all of the following except: Widening of the joint space Wearing away of articular cartilage Formation of bone spurs Synovial membrane thickens

Widening of the joint space

Patients with stress incontinence often report urine loss: With lifting At night Associated with a strong sensation of needing to void As dribbling after voiding

With lifting

Which of the following best describes the presentation of a patient with OA? Worst symptoms in weight-bearing joints later in the day Symmetrical early morning stiffness Sausage-shaped digits with associated skin lesions Back pain with rest and anterior uveitis

Worst symptoms in weight-bearing joints later in the day

In caring for a pt with DM, microalbuminuria measurement should be obtained: Annually if urine protein is present Periodically in relationship to glycemic control Yearly With each office visit r/t DM

Yearly

Risk factors for UTI in women include: A postvoid wiping back to front Low perivaginal lactobacilli colonization Hot tub use Wearing snug-fitting pantyhose

Low perivaginal lactobacilli colonization

Which of the following characteristics applies to type 2 DM? Major risk factors are heredity and obesity Pear-shaped body type is commonly found Exogenous insulin is needed for control of disease Physical activity enhances insulin resistance

Major risk factors are heredity and obesity

Osteoporosis is more common in individuals: With DM2 On long-term system corticosteroid tx Who are obese Of African ancestry

On long-term system corticosteroid tx

Leakage occurring about equally as often with physical activity as with a sense of urgency.

Mixed (stress and urgency)

A 14M presents with acne consisting of 25 comedones and 20 inflammatory lesions with no nodules. This patient can be classified as having: Mild acne Moderate Severe Very severe

Moderate

As part of the eval of pts with OA, the NP anticipates finding: Anemia of chronic disease Elevated CRP level No disease-specific lab abnormalities Elevated antinuclear antibody (AN) titer

No disease-specific lab abnormalities

Common triggers for anaphylaxis include exposure to certain types of all of the following except: Medications Food Pet dander Insect bites

Pet dander

Criteria for the diagnosis of type 2 DM include: Classic symptoms regardless of fasting plasma glucose measurement Plasma level of 126 as a random measurement 2hr glucose of 156 after a 75g anhydrous glucose load Plasma level of 126 or greater after an 8 hour or greater fast on more than 1 occasion

Plasma level of 126 or greater after an 8 hour or greater fast on more than 1 occasion

All of the following can negatively impact perivaginal lactobacilli colonization except: Recent antimicrobial use Exposure to the spermicide nonoxynol-9 Estrogen deficiency Postcoital voiding

Postcoital voiding

Which part of the prostate is readily palpable during a DRE? Anterior lobe Median lobe Lateral lobes Posterior lobe

Posterior lobe

The meglitinide analogues are particularly helpful adjuncts in DM2 to minimize risk of: Fasting hypoglycemia Nocturnal hyperglycemia Postprandial hyperglycemia Postprandial hypoglycemia

Postprandial hyperglycemia

Which of the following meds is a helpful tx option for relief of tremor and tachycardia seen with untreated hyperthyroidism? Propranolol Diazepam Carbamazepine Verapamil

Propranolol

Finasteride (Propecia) and dutasteride (Avodart) are helpful in the TX of BPH because of their effect on: Bladder contractility Prostate size Activity at select bladder receptor sites Bladder pressure

Prostate size

To help prevent meniscal tear, you advise: Limiting participation in sports Quad-strengthening exercises Using a knee brace Applying ice to the knee before exercise

Quad-strengthening exercises

In caring for a pt with OA of the knee, you advise that: Straight-leg raising should be avoided Heat should be applied to painful joints after exercise Quad-strengthening exercises should be performed Physical activity should be avoided

Quad-strengthening exercises should be performed

Psoriatic lesions arise from: Decreased skin exfoliation Rapid skin cell turnover, leading to decreased maturation and keratinization Inflammatory changes in the dermis Lichenification

Rapid skin cell turnover, leading to decreased maturation and keratinization

A 52 y/o female comes to the clinic for a health maintenance visit. Past medical history is significant for HTN, hyperlipidemia, and diabetes. Her BMI from 3 months ago was 31.5. FOr the past 3 months, she has restricted calories and increased her aerobic activity. Current medications are lisinopril, aspirin, atorvastatin, and metformin. Her pulse is 76, BP 138/76 and BMI 31. Her physical exam is normal. Her HbA1c is 7.6%. The patient is frustrated as her weigh has not improved significantly. Involvement of which of the following individuals would me most appropriate in this care of this patient? Cardiologist Endocrinologist Physical therapy Registered nutritionist Social worker

Registered nutritionist

Which of the following statements is true concerning elder maltreatment? This problem is found mainly in families of lower SES. An elderly adult who is being mistreated usually seeks help Routine screening is indicated as part of the care of an older adult In most instances of elder maltreatment, a predictable cycle of physical violence directed at the older adult followed by a period of remorse on the part of the perpetrator is the norm

Routine screening is indicated as part of the care of an older adult

Which of the following is a not a gram-negative organism? E coli K pneumoniae P mirabilis S saprophyticus

S saprophyticus

A positive ANA test is a sensitive marker for the presence of: Hyperparathyroidism SLE Kawasaki dz Leukocytosis

SLE

The daughter of a 76F expresses concern regarding her mother's refusal of assistance in everyday activities. The mom lives by herself and is often found with poor hygiene and reports eating one small meal a day. She also has poor adherence to her current med regimen. This represents: Abandonment Self-neglect Early-onset dementia Psychological abuse

Self-neglect

A 30 y/o woman with type 2 diabetes uses regular insulin and intermediate-acting insulin in the morning and evening. She denies any changes in her diet or any illness, but she recently started attending aerobic classes in the afternoon. Her fasting blood glucose level before breakfast is now elevated. Which of the following is best described? Somogyi phenomenon Dawn phenomenon Raynaud's phenomenon Insulin resistance

Somogyi phenomenon

A patient with a lumbosacral strain will typically report: Numbness in the extremities Stiffness, spasm, and reduced ROM "electric" sensation running down one or both legs Pain at its worst when in sitting position

Stiffness, spasm, and reduced ROM

Leakage with performing some physical activity, such as coughing, lifting, or exercise.

Stress only or stress predominant

A male patient has type 2 diabetes mellitus and a "sensitive stomach". Which medication is least likely to cause him gastrointestinal distress? Naproxen sodium (anaprox) Aspirin (Bayers aspirin) Eryhtomycin (e-mycin) Sucralfate (carafate)

Sucralfate (carafate)

Patients with rosacea are recommended to use daily: Sunscreen Astringents Exfoliant Antimicrobial cream

Sunscreen

Treatment options in generalize psoriasis vulgaris include all of the following except: Psoralen with ultraviolent A light (PUV A) therapy Methotrexate Cyclosporine Systemic corticosteroids

Systemic corticosteroids

What is the treatment for Urge incontinence?

anticholinergics (like fesoterodine (toviaz))

A 28 y/o male presents with a skin lesion on his left upper arm that has been present for as long as he can remember, however, just recently it has been changing in size and color. The lesion does not itch or cause pain. On the physical exam, you note an 8mm round symmetrical patch that is brown with areas of black. The lesion is flat without crusting or ulceration. What is the next step in his management? a. Deep shave biopsy b. Excisional biopsy c. Monitoring of the lesion with f/u in 6 months d. Punch biopsy e. Superficial shave biopsy

b. Excisional biopsy

A 25 y/o female presents with a skin lesion on her upper right back that has been present for years, but recently changed in color and size. She reports it is not itchy or painful and does not bleed easily. She is a runner and runs outside often. She wears sunscreen in the summer with SPF 30. On exam the lesion is 7mm flat patch, irregular and asymmetric in appearance, and has a mixture of brown and black shades. Based on this patient's history and exam, which finding has the highest specificity for melanoma. a. Asymmetric appearance b. Color variation c. Diameter d. Evolving character e. History of frequent sun exposure

d. Evolving character

Fine tremors Is this a finding in (1) hyperthyroidism, (2) hypothyroidism or (3) both?

hyperthyroidism

Dry skin Is this a finding in (1) hyperthyroidism, (2) hypothyroidism or (3) both?

hypothyroidism

Hypoactive deep tendon reflexes (DTR) Is this a finding in (1) hyperthyroidism, (2) hypothyroidism or (3) both?

hypothyroidism

Menorrhagia Is this a finding in (1) hyperthyroidism, (2) hypothyroidism or (3) both?

hypothyroidism

The use of which of the following meds can induce thyroid dysfunction? Sertraline Venlafaxine Bupropion lithium

lithium

A 46 y/o female with diabetes and obesity presents to the office for a routine f/u. She is currently asymptomatic and takes metformin. She would like to lose weight. On the physical exam her HR is 76, BP 136/72, BMI 42. She asks if there is anything she could be doing to improve her health. Which of the following is the most appropriate counseling statement for this patient? "I think you should try to lose weight." "We need to refer you to the bariatric surgeon given your diabetes and high BMI." "We will work together on trying to achieve a 5-10% weight loss in weight over the next six months." "You should exercise more in order to lose weight." "You should attempt to lose enough weight to achieve a normal BMI."

"We will work together on trying to achieve a 5-10% weight loss in weight over the next six months."

A 75-year-old man is admitted to the orthopedic unit after surgery to repair a fractured right femur. He fell off a 6′ ladder, and approximately 6 hours passed before he was found, lying on his back in the yard. He had surgery 3 hours after arrival at the emergency department. The admitting nurse on the orthopedic unit documents a 7 × 10 cm purple-maroon discoloration at the sacral-coccygeal area; skin is intact. The discoloration is nonblanching and symmetric. It is documented as ecchymosis from a traumatic fall. Which one of the following pressure injury designations describes the ecchymotic area? (A) Deep-tissue injury (B) Stage 1 (C) Stage 2 (D) Stage 3

(A) Deep-tissue injury

Mr. Johnson is a 45-year-old with diabetes who is homeless. He presents with tender fluctuant nodules on the skin around his beard. He has been previously treated with benzoyl peroxide 5% with minimal response. You order warm compresses and? (A) Dicloxacillin 500 mg qid for 10 days (B) Trimethoprim/sulfamethoxazole daily for 5 days (C) Acyclovir topical (D) Clotrimazole 1% bid for 10 days

(A) Dicloxacillin 500 mg qid for 10 days

With fluoroquinolone use, length of antimicrobial therapy during uncomplicated pyelonephrtitis is typically: 5 days 1 week 2 weeks 3 weeks

1 week

Older adults are more likely to have symptomatic hypoglycemia episodes than younger people. Which of the following symptoms of hypoglycemia is less common in older adults than younger adults? 1-Anxiety 2- Delirium 3- Dizziness 4- Seizures 5- Weakness

1-Anxiety

which urine culture result is needed to confirm a UTI in an asymptomatic woman who has not had recent use of a urinary catheter? 10^2 cfu/mL or more 10^3 or more 10^4 or more 10^5 or more

10^5 or more

Which of the following is considered as a precursor lesion of squamous cell skin cancer? 1- Atopic dermatitis 2- Actinic keratosis 3- Seborrheic keratosis 4- Nevi

2- Actinic keratosis

All of the following pharmacologic agents are indicated for the treatment of plaque psoriasis, except: 1- Topical corticosteroids 2- Vitamin D analogs 3- UV light therapy 4- Oral antibiotics

4- Oral antibiotics

Elder maltreatment is considered to be underreported, with an estimated ___ cases going unreported to each one case that is reported. 3 4 5 6

5

You see a 33M w/a minor dog bite with a superficial wound. The dog is current with immunizations. You consider __ of dog bites become bacterially infected. 5% 20% 50% 75%

5%

Recommendation a1c goal in 79F with 20y h/o DM2 who has difficulty walking, uses a walker, and has an EF of 35% and h/o of HF should be equal to or less than: 7% 7.5 8 8.5

7.5

After use, the onset of action of lispro 9humalog) occurs in: <30 mins About 1 hour 1-2 hours 3-4 hours

<30 mins

A male patient with diabetes asks the clinician why he needs to check his blood sugar at home even when he feels good. Which response by the clinician would be most appropriate? A. "Control of glucose will help postpone or delay complications." B. "Regularly checking blood sugar will help you." C. "Monitoring glucose will promote a sense of connectedness." D. "Because you know it is the right thing to do for your disease."

A. "Control of glucose will help postpone or delay complications."

Which statement made by a patient with type 1 diabetes indicates successful teaching regarding general guidelines for exercise? A. "I will not exercise if my blood sugar is 315." B. "I should just check my blood glucose after exercise." C. "I should eat more carbohydrates if my blood glucose is less than 200." D. "I will exercise when my urine ketones are positive but blood sugar is 260."

A. "I will not exercise if my blood sugar is 315."

You have detected the presence of crepitus on examination of a patient with a musculoskeletal complaint. Additionally, there is limited range of motion (ROM) with both active and passive movement. These findings suggest that the origin of the musculoskeletal complaint is: A. Articular B. Inflammatory C. Nonarticular D. Noninflammatory

A. Articular

The clinician has instructed Sirius, a 23-year-old patient with low back strain, to use NSAIDs to manage his symptoms of pain and discomfort. Which of the following statements would be most appropriate when teaching Sam about the use of NSAIDs? A. "You should start with the lowest dose that is effective in managing your pain, because long-term use of NSAIDs can result in gastrointestinal (GI) disorders such as ulcers and hemorrhage." B. "You should start with the lowest dose that is effective in managing your pain to avoid developing tolerance to the medication." C. "You should take the maximum recommended dose of NSAIDs so that you will not need to take narcotics to control your pain." D. "It is important to take NSAIDs on an empty stomach in order to increase absorption."

A. "You should start with the lowest dose that is effective in managing your pain, because long-term use of NSAIDs can result in gastrointestinal (GI) disorders such as ulcers and hemorrhage."

A 55 y/o female pt presents w/ symptoms of a UTI. A clean catch midstream urine specimen is obtained and sent to the laboratory for culture and sensitivity testing. Which of the following results is indicative of an uncomplicated UTI? A. 10^5 CFU/ mL of one organism B. 10^5 CFU/mL of one or more organism C. 10^3 CFU/mL of enterobacteriaceae D. 10^3 CFU/mL of gram-negative Enterobacteriaceae

A. 10^5 CFU/ mL of one organism

4. An 88-year-old patient is seen in her urogynecologist's office for increasing urinary incontinence. Her symptoms have been present for three months. She urinates seven to eight times a day and is frequently rushing to the bathroom. She never has any leakage when she coughs or sneezes. She has no vaginal pressure but does endorse vaginal irritation and dryness. Past medical history is significant for mild dementia, essential hypertension (HTN), and hyperlipidemia. She had a total abdominal hysterectomy when she was 40 years old due to uterine fibroids. Medications include lisinopril, simvastatin, and fish oil. Vital signs are normal. The genitourinary examination is completed. The labia appear normal. The vagina has whitish-pink coloration and is dry. There is minimal ruggation present. There are no prolapses visualized. Pelvic floor strength is normal and she has mild discomfort from skin irritation during the vaginal exam. Urinalysis is normal. Post-void residual is 5 ml. Which of the following underlying conditions most likely contributes to her urinary symptoms? Choose the single best answer. A. Atrophic vaginitis B. Cystocele C. Hyperestrogenism D. Pelvic floor muscle weakness E. Vulvar dermatitis

A. Atrophic vaginitis

3. An 80-year-old patient is admitted to the hospital for an acute right-hip fracture after a fall. She undergoes surgical fixation and is discharged to a rehabilitation facility for continued therapy. Prior to admission, she was independent in her activities of daily living (ADLs), ambulated without any assistive devices, and lived alone in a condominium. She has a medical history of osteoporosis and hypothyroidism. Her medications prior to admission include calcium, vitamin D3, alendronate, and levothyroxine. During hospital admission, she was started on oxycodone three times daily for pain control, which is being continued in rehab. She has progressed well in rehab and has occasional concerns of pain in her right hip. She has resumed a normal diet; her last bowel movement was three days ago. She is quite distressed about a new onset of urinary leakage over the last week. She has no dysuria or hematuria. She feels like she is always rushing to the bathroom and rarely makes it in time now. On physical examination, vital signs are normal. Bowel sounds are hypoactive. There are no palpable abdominal masses, no suprapubic tenderness, and no costovertebral angle tenderness. The gynecologic examination is unremarkable. Her gait is swift and assisted with a two-wheeled walker. Balance is intact. Surgical incisions of her right hip have healed well. A urinalysis is obtained and shows mild leukocyte esterase. Post-void residual is 150 ml. What is the likely etiology of her urinary incontinence? Choose the single best answer. A. Constipation B. Decreased mobility C. Pelvic surgery D. Urinary tract infection E. Vaginal atrophy

A. Constipation

All of the following are common sites of fracture in pts with osteoporosis except: Proximal femur Distal forearm Vertebrae Clavicle

Clavicle

Surgical intervention in BPH should be considered w/all of the following except: Recurrent UTI Bladder stones Persistent obstruction despite medical therapy Acute tubular necrosis

Acute tubular necrosis

All of the following meds are recommended as possible 1st-line meds for HTN with DM2 for a European man except: Thiazide diuretic Calcium channel blocker Alpha-adrenergic receptor antagonist Angiotensin receptor blocker

Alpha-adrenergic receptor antagonist

Which diagnostic test is the diagnostic gold standard for patients that have tendonitis and have failed conservative treatment? A. Plain x-ray films B. Magnetic resonance imaging (MRI) C. Computed tomography (CT) scan D. Surgical exploration

B. Magnetic resonance imaging (MRI)

John is a 16-year-old boy who presents to the emergency room after hurting his knee in a football game. He described twisting his knee and then being unable to extend it completely. John tells the clinician that he heard a pop when the injury occurred and has been experiencing localized pain. The clinician suspects a meniscal tear. Which test would be most appropriate to assess for the presence of a meniscal tear? A. Valgus stress test B. McMurray circumduction test C. Lachman test D. Varus stress test

B. McMurray circumduction test

An 80-year-old patient presents to an urogynecology clinic with a complaint of urinary incontinence. She has been suffering from her symptoms for about one year. She has to wear a sanitary pad for leak protection; she changes it three times a day. She has a history of hysterectomy. Her other pertinent medical history includes essential hypertension, venous insufficiency, and constipation. Medications include amlodipine and senna. Vital signs are normal. She is a well-appearing, mildly obese elderly female. Abdominal exam is unremarkable. On genitourinary exam, there is mild vaginal atrophy noted and mild cystocele. Results of laboratory/diagnostic studies show a normal urinalysis. You diagnose her with urge urinary incontinence. Which of the following is the best next step in the management of this patient? Choose the single best answer. A. Oxybutynin B. Pelvic floor therapy C. Pessary placement D. Referral to urology E. Surgical intervention of cystocele

B. Pelvic floor therapy

Debbie is a 43-year-old female being evaluated for a wrist injury. The clinician is assessing for median nerve compression by having Debbie maintain forced flexion of her wrist for 1 minute with the dorsal surface of each hand pressed together. Which of these tests did the clinician just perform? A. Allen's test B. Phalen's test C. Tinel's sign D. Finkelstein's test

B. Phalen's test

A patient with type 1 diabetes has diabetic ketoacidosis. Which first-line treatment should the clinician prescribe? A. NPH insulin B. Regular insulin C. IV normal saline D. Thiazolidinedione

B. Regular insulin

A new patinet, who is 54 y/o athlete with a history of hyperlipiedma, is seen for new onset of generalized myalgia, muscle weakness, oliguria, and tea-colored urine for the past 24 hours. He recently completed running a marathon. He is on a statin and reports taking ibuprofen sevveral times to treat the muscle pain. Which diagnosis is most likely? A. Hyperthyroidism B. Rhabdomyolysis C. Pyelonephritis D. Viral influenza

B. Rhabdomyolysis

Which of the following classes of meds is commonly recommended as part of first-line therapy in the newly dx'ed DM2? Alpha -glucosidase inhibitor Meglitinide Thiazolidinedione Biguanide

Biguanide

Long-term effects of SLE can include all of the following except: Birth defects when occurring in a pregnant female Kidney failure Avascular necrosis Pericarditis

Birth defects when occurring in a pregnant female

Which of the following is inconsistent w/the description of BPH? Obliterated median sulcus Size larger than 2.5 cm x 3 cm Sensation of incomplete emptying Boggy gland

Boggy gland

Possibly consequences of excessive levothyroxine use include: Bone thinning Fatigue Renal impairment Constipation

Bone thinning

A 64F presents with urge incontinence and has not been able to tolerate tx with anticholinergic agents. You recommend the use of which of the following? (more than 1 can apply) Botulinum toxin injections Fesoterodine fumarate (toviaz) Mirabegron (myrbetriq) Finasteride (proscar)

Botulinum toxin injections & Mirabegron (myrbetriq)

Deformity of the proximal interphalangeal joints found in an elderly pt with OA is known as: Heberden nodes Bouchard nodes Hallus valgus Dupuytren contracture

Bouchard nodes

The major risk factor for thyroid cancer is: A ] Inadequate iodine intake B ] Presence of a goiter C ] Exposure to radiation D ] Smoking

C ] Exposure to radiation

Which is the most effective method to differentiate a benign from a malignant thyroid nodule: A ] Magnetic resonance imaging (MRI) B ] Ultrasound C ] Fine-needle aspiration biopsy D ] Radioactive iodine scan

C ] Fine-needle aspiration biopsy

Mr. Jackson is a 65-year-old man recently diagnosed with osteoarthritis. The clinician has explained to Mr. Jackson that the goals for managing osteoarthritis include controlling pain, maximizing functional independence and mobility, minimizing disability, and preserving quality of life. Mr. Jackson explains to the clinician that his first choice would be to use complementary therapies to control his condition and asks what therapies are most effective in treating osteoarthritis. What would be the most appropriate response from the clinician? A. "Complementary therapies should be considered only if surgical interventions are not successful." B. "I am unfamiliar with the available complementary therapies for osteoarthritis and prefer to discuss more mainstream treatments, such as NSAIDs and physical therapy, to manage your condition." C. "I would be happy to discuss all the treatment options available to you. Complementary therapies, such as acupuncture, acupressure, and tai chi, are being studied for use in the treatment of osteoarthritis and acupuncture can be used and is safe and well tolerated." D. "It would be crazy to use complementary therapies to treat such a serious condition."

C. "I would be happy to discuss all the treatment options available to you. Complementary therapies, such as acupuncture, acupressure, and tai chi, are being studied for use in the treatment of osteoarthritis and acupuncture can be used and is safe and well tolerated."

A 35-year-old female presents with urgency, frequency, and pain when she urinates. Other associated symptoms include left-lower-back pain, fever, chills, nausea, and vomiting. She has no medical problems and does not take any medications. On physical examination her vital signs show her temperature is 38.9 C (102 F), blood pressure is 90/55 mmHg, pulse is 120 beats/minute, respiratory rate is 18 breaths/minute, and oxygen saturation is 98% on ambient air. She has left flank tenderness and lower abdominal tenderness. Laboratory studies show an elevated white blood cell count, but her basic metabolic panel is unremarkable. Urinalysis reveals pyuria with white blood cell casts. Blood and urine cultures are sent. In addition to intravenous fluids and hospital admission, which of the following is the most appropriate next step? The best option is indicated below. Your selections are indicated by the shaded boxes. A. Azithromycin B. Cefpodoxime C. Ceftriaxone D. Moxifloxacin E. Vancomycin and piperacillin/tazobactam

C. Ceftriaxone

The patient has metabolic syndrome. The patient is at increased risk for which condition? A. Anxiety disorder B. Gallbladder disease C. Diabetes mellitus D. Hyperparathyroidism

C. Diabetes mellitus

A patient is 66 inches in height, weighing 200 lbs., and newly diagnosed with type 2 diabetes mellitus (DM). The A1c is 7.1%. What is the best initial treatment? A. No treatment at this time B. Diet and exercise C. Diet, exercise, and metformin D. Diet, exercise, and exogenous insulin

C. Diet, exercise, and metformin

An 18 y/o woman is complaining of R ankle pain. She reports twisting her ankle while jogging the previous day. She states that she "rolled inward on her ankle. During the physical exam, the NP finds a markedly swollen ankle with several areas of ecchymosis. There is tenderness on the tip of the lateral malleolus. The joint is tender on palpation. The patient has problems w/ weight bearing and walking. What time of sprain does this patient have? A. Grade I B. Grade II C. Grade III D. Grade IV

C. Grade III

Which of the following signs or symptoms indicate an inflammatory etiology to musculoskeletal pain? A. Decreased C-reactive protein B. Hyperalbuminemia C. Morning stiffness D. Weight gain

C. Morning stiffness

A 50-year-old male with a history of hypertension presents with urgency, frequency, and pain when he urinates. He reports no fever, but has chills, lower abdominal pain, cloudy urine, and nausea. He notes he had a similar presentation about a year ago that resolved with antibiotics. He states that he has a weak urinary stream and a sensation that his bladder is full even after urinating. His current medications are acetaminophen and amlodipine. On physical examination his vital signs are normal, but his examination is limited due to severe suprapubic tenderness. Laboratory studies show an elevated white blood cell count, normal basic metabolic panel, and negative gonorrhea and chlamydia testing. His urinalysis is consistent with a urinary tract infection (UTI). In addition to antibiotic treatment, which of the following is the most appropriate next step? A. Cystoscopy B. No further management C. Post-void residual bladder volume D. Repeat the urinalysis and urine culture E. Transurethral resection of the prostate

C. Post-void residual bladder volume

Which of the following is a risk factor for overuse syndrome with tendonitis? A. Body mass index <18 B. Hypothyroidism C. Rheumatoid arthritis D. Cardiac disease

C. Rheumatoid arthritis

A 62F p/w 2 days after a big bite - revealing a warm, red, edematous area with sharply demarcated borders. No fever. This is most likely: Contact dermatitis An allergic reaction Cellulitis Erysipelas

Cellulitis

An example of a 1st-line therapeutic agent for the tx of pyelonephritis is: Amoxicillin with clavulanate Trimethoprim-sulfamethoxazole Cipro Nitrofurantoin

Cipro

You see a 70F in a walk-in center w/a chief complaint of increase urinary frequency and dysuria. UA reveals pyuria and positive nitrites. She mentions she has a "bit of kidney trouble, not too bad." Recent evaluation of renal status is unavailable. In considering antimicrobial tx, you rx: Nitrofurantoin Fosfomycin Cipro Doxycycline

Cipro

A 25-year-old female presents for advice on contraception methods. She has a history of migraine with aura, but states she has not had a migraine in six months. She takes non-steroidal antiinflammatory drugs as needed for the migraines. She is working at her "dream job", although she often has to work 12- to 14-hour days. She lives with her boyfriend, and they will be getting married in two months. She would like to start having children in about two to three years. She has normal monthly menstrual cycles that last about four to five days. Her last menstrual period was two days ago. She states no alcohol or illicit drug use, but admits that she often smokes 5-10 cigarettes a week and is trying to quit. Physical examination and vital signs are unremarkable. She is requesting the most-effective contraception method against pregnancy that can accommodate her busy work schedule. She is concerned about weight gain. What would be the most appropriate contraception for this patient? A. Condoms B. Depot medroxyprogesterone acetate C. Estrogen/progestin contraceptive pills D. Levonorgestrel intrauterine device E. Post-coital contraceptives

D. Levonorgestrel intrauterine device

All of the following groups are considered complicated UTI, except: A. Males B. Infants C. Pregnant Women D. Middle Aged Women

D. Middle Aged Women

Chris is a 28-year-old male who complains of lower back pain that began 3 days ago. The pain is worse when he stands or bends, and it is somewhat relieved when he sits. The clinician performs the straight-leg raise test and it is negative. Plain film x-ray is positive. Which diagnosis is most likely? A. Osteoarthritis B. Spinal stenosis C. Scoliosis D. Muscle strain

D. Muscle strain

Which "P" is a component of "the 3 P's" of diabetes mellitus? A. Pain B. Paresthesia C. Pallor D. Polyphagia

D. Polyphagia

At this visit, you would consider the following treatments for her urinary symptoms with the EXCEPTION of those listed below: Choose the single best answer. A. Decrease her caffeine and fluid intake, especially in the evenings B. Pelvic floor muscle exercises C. Scheduled voiding D. Start an antimuscarinic bladder medication, like oxybutynin E. Stop TYLENOL® PM

D. Start an antimuscarinic bladder medication, like oxybutynin

You see an 82F with early-onset dementia and urge incontinence. Which of the following meds is least likely to contribute to worsening mental status? Oxybutynin (Ditropan) Tolterodine (Detrol) Darifenacin (enablex) Solifenacin succinate (vesicare)

Darifenacin (enablex)

Which of the following joints is most likely to be affected by OA? Wrists Elbows Metacarpophalangeal joint Distal interphalangeal joint

Distal interphalangeal joint

Periodic routine screening for hypothyroidism is indicated in the presence of which of the following clinical conditions? Digoxin use Male gender Down syndrome Alcoholism

Down syndrome

A 72 y/o patient is brought to the ED because of a recent seizure witnessed by her son. He told EMS that just prior to her seizure his mother had appeared confused, slow and shaky. Medical history is significant for diabetes, HTN and retinopathy. She was seen in the local urgent care this week and is being treated with Bactrim for a UTI. Her usual medication includes glyburide 5 mg daily and lisinopril 5 mg daily. The patient received care in the EMS ambulance and arrived at the ED with normal vitals. The paperwork of what was done for her on the ambulance is not available. She is no longer experiencing a seizure, but her cognition remains slow, and she is uncertain of the day and the year. She cannot recount what occurred. Her exam EKG and basic lab studies are WNL on arrival. Reviewing the patient's history and recent events, which of the following best explains the precipitating factor of her recent seizure? 1. Amplified side effect 2. Disease-disease interaction 3. Drug-disease interaction 4.Drug-drug interaction 5. Physiologic change with aging

Drug-drug interaction

Common adverse effects of musculotropic relaxants (i.e. fesoterodine (toviaz)) used in the tx of urinary incontinence include: Dry mouth and constipation Nausea Headaches Syncope

Dry mouth and constipation

The preferred screening test for osteoporosis is: Quantitative US measurement Dual-energy XR absorptiometry Qualitative CT Wrist, spine, and hip radiographs

Dual-energy XR absorptiometry

The most likely causative organism in community-acquired UTI in women during the reproductive years is: Klebsiella Proteus mirabilis E coli staphylococcus saprophyticus

E coli

Which of the following best describes the physical activity recommendations such as brisk walking for a 55F with new DM2? A- Goal should be for a total increased physical activity of 150 min/wk or more B- Increased physical activity is recommended at least 3x/wk with no more than 48h without exercise C- Some form of resistance exercise such as lifting dumbbells or using an exercise band should be done 2x/week D- Vigorous aerobic or resistance activity is potentially contraindicated in the presence of proliferative or severe nonproliferative reinopathy because of possible risk of vitreous hemorrhage or retinal detachment E- all of the above

E- all of the above

2.A 90-year-old patient comes to the clinic because of a nine-month history of urinary incontinence. She explains her incontinence as typically occurring when she hears water running or cannot get to the bathroom in time. She also experiences urinary leakage when she gets up from lying down and when she coughs. She lives a very sedentary lifestyle and ambulates with a two-wheeled walker. Medical history is significant for osteoporosis, hypertension, constipation, and primary osteoarthritis of her knees. Surgical history includes total left hip replacement, appendectomy, and total abdominal hysterectomy. Medications are alendronate, amlodipine, and acetaminophen. Vital signs are normal. Physical examination shows a soft abdomen with no abdominal masses. Genitourinary examination shows moderate vaginal atrophy with a mild to moderate cystocele and decreased strength of pelvic floor muscles. The patient appears frail. Results of laboratory/diagnostic studies show a post-void residual of 50 ml and normal urinalysis. Which of the following is contributory to her urinary incontinence symptoms? Choose the single best answer. A. Chronic constipation B. Decreased estrogen production C. History of hysterectomy D. Pelvic floor weakness E. All of the above

E. All of the above

A 45-year-old female presents with urgency, frequency, and pain when she urinates. She also has left-lower back pain, fever, chills, nausea, and vomiting. She has a history of type 2 diabetes mellitus and takes metformin. On physical examination, her vital signs show her temperature 38.9 C (102 F), blood pressure is 90/55 mmHg, pulse is 120 beats/minute, respiratory rate is 18 breaths/minute, and oxygen saturation is 98% on ambient air. She has left flank tenderness and lower abdominal tenderness. Laboratory studies showed an elevated white blood cell count. Urinalysis showed pyuria with white blood cell cast. Blood and urine cultures were sent. She is admitted to the hospital and started on intravenous fluids and ceftriaxone. Urine culture shows E. coli sensitive to cephalosporins. Despite three days of antibiotics, she remains febrile with a peak temperature of 38.9 C (102 F). Her abdominal pain persists, albeit slightly better, and she continues to have nausea and vomiting. Which of the following is the most appropriate next step? A. Add vancomycin B. Change ceftriaxone to another class of antibiotic C. Consult urology D. Obtain a chest radiograph E. Obtain a renal ultrasound

E. Obtain a renal ultrasound

RA disease progression is typically evaluated using all of the following approaches except: XR MRI Echosonography US

Echosonography

Increased risk of thyroid disorder is found in individuals who are: Obese Hypertensive Treated with systemic corticosteroids Elderly

Elderly

Which of the following tests is most specific to the dx of RA? Elevated levels of rheumatoid factor Abnormally high ESR Leukopenia Positive ANA titer

Elevated levels of rheumatoid factor

Which of the following best describes Seborrheic dermatitis lesions? Flaking lesions in the antecubital and popliteal spaces Greasy, scaling lesions in the nasolabial folds Intensely itchy lesions in the groin folds Silvery lesions on the elbows and knees

Greasy, scaling lesions in the nasolabial folds

Cardiovascular effects of hyperinsulinemia include: Decreased renal sodium reabsorption Constricted circulating volume Greater responsiveness to angiotensin II Diminished sympathetic activation

Greater responsiveness to angiotensin II

The most common causative organisms in cellulitis are: E coli and h influenzae Bacteroides species and other anaerobes Group A bet-hemolytic streptococci and s aureus Pathogenic viruses

Group A bet-hemolytic streptococci and s aureus

A 38F with HIV p/w a painful, itchy rash over her trunk. Exam reveals linear vesicular lesions that do not cross the midline and are distributed over the posterior thorax. This presentation is most c/w: Herpes zoster Dermatitis herpetiformis Molluscum contagiosum Impetigo

Herpes zoster

Risk factors for prostate cancer include all of the following except: African ancestry History of genital trauma Family history of prostate cancer High-fat diet

History of genital trauma

Which of the following is not true concerning the effects of exercise and insulin resistance? Approximately 80% of the body's insulin-mediated glucose uptake occurs in skeletal muscle With regular aerobic exercise, insulin resistance is reduced by about 40% The insulin resistance-reducing effects of exercise persist for 48h after the activity Hyperglycemia can occur as a result of aerobic exercise

Hyperglycemia can occur as a result of aerobic exercise

Clinical disorders that increase the risk for osteoporosis include all but: RA Celiac dz Hyperlipidemia Hyperprolactinemia

Hyperlipidemia

Which of the following is an unlikely consequence of untreated metabolic syndrome and insulin resistance in a woman of reproductive age? Hyperovulation Irregular menses Acne Hirsutism

Hyperovulation

Common clinical manifestations of anaphylaxis can include all of the following except: Upper airway edema Itch without rash Dizziness with syncope Hypertension

Hypertension

Exophthalmos Is this a finding in (1) hyperthyroidism, (2) hypothyroidism or (3) both?

Hyperthyroidism

The following findings are found in which condition? heat intolerance, smooth, silky skin, frequent, low-volume, loose stools, amenorrhea or oligomenorrhea, hyperreflexia with a characteristic "quick out-quick back" action at the patellar reflex, proximal muscle weakness, tachycardia with hypertension

Hyperthyroidism

The following findings are found in which condition? Secondary hypertriglyceridemia, coarse, dry skin, menorrhagia, hyporeflexia with a characteristic slow relaxation phase, the "hung-up" reflex, coarse hair with tendency to break easily, thick, dry nails, constipation

Hypothyroidism

In treatment of acne, which lesions respond best to topical antibiotic therapy? Open comedones Cysts Inflammatory lesions Superficial lesions

Inflammatory lesions

You see a 28M who is having an anaphylactic reaction following a bee sting and is experiencing trouble breathing. Your initial response it to give: Oral antihistamine Injectable epinephrine Supplemental O2 Vasopressor therapy

Injectable epinephrine

Which is most accurate regarding cellulitis? Insect bites, abrasion, or other skin trauma can be the origin of cellulitis Cellulitis most often occurs on the chest and abdomen Necrosis is a common complication of cellulitis Cellulitis often occurs spontaneously w/o any identifiable skin wound

Insect bites, abrasion, or other skin trauma can be the origin of cellulitis

The use of which med has the potential for causing the greatest reduction in hgba1c? Biguanide Thiazolidinedione Sulfonylurea Insulin form

Insulin form

Which of the following statements best describes the Somogyi effect? Insulin-induced hypoglycemia triggers excess secretion of glucagon and cortisol, leading to hyperglycemia Early morning elevated blood glucose levels result in part from growth hormone and cortisol-triggering hepatic glucose release Late evening hyperglycemia is induced by inadequate insulin dose Episodes of postprandial hypoglycemia occur as a result of inadequate food intake

Insulin-induced hypoglycemia triggers excess secretion of glucagon and cortisol, leading to hyperglycemia

You see a 28M who was in a fight with a man and bitten. The left arm reveals an open wound. Your next best action is: Obtain a culture and sensitivity of the wound site Refer for rabies prophylaxis Irrigate the wound and debride as needed Close the wound with adhesive strips

Irrigate the wound and debride as needed

Hemorrhagic cystitis is characterized by: Irritative voiding symptoms Persistent microscopic hematuria The presence of hypertension Elevated creatinine and BUN levels

Irritative voiding symptoms

Oral antifungal treatment options for onychomycosis include all of the following except: Itraconazole Fluconazole Metronidazole Terbinafine

Metronidazole

Which one of the following findings is associated with diabetic retinopathy? AV nicking Copper wire arterioles Flame-shaped hemorrhages Microaneurysms

Microaneurysms

In diagnosing onychomycosis, the NP considers that: Mails often have a single midline groove Pitting is often seen Microscopic exam reveals hyphae Beau lines are present

Microscopic exam reveals hyphae

Most episodes of low back pain are caused by: An acute precipitating event Disk herniation Muscle or ligamentous strain Nerve impingement

Muscle or ligamentous strain

All of the following diagnostic findings are expected in a pt with SLE except: Elevated ESR Anemia Negative ANA test Proteinuria

Negative ANA test

The most commonly reported form of elder maltreatment is: Physical abuse Sexual exploitation Financial exploitation Neglect

Neglect

You see a 34yo woman with an uncomplicated UTI. She is otherwise healthy but reports having a sulfa allergy. Appropriate therapy would be: TMP-SMX Amoxicillin Azithromycin Nitrofurantoin

Nitrofurantoin

Diagnostic criteria for metabolic syndrome includes: Obligatory finding of persistent hyperglycemia Notation of ethnic-specific waist circumference measurements Documentation of microalbuminuria Family h/o DM2

Notation of ethnic-specific waist circumference measurements

You see a 52F bitten by a rat with a 1 cm deep wound oozing bright red blood. Treatment should include standard wound care plus: Rabies immune globulin Rabies vaccine Oral cipro Oral amoxicillin-clavulanate

Oral amoxicillin-clavulanate

Radiographic findings of OA of the knee often reveal: Microfractures Decreased density of subchondral bone Osteophytes No apparent changes to the joint structure

Osteophytes

Leakage without physical activity and without a sense of urgency.

Overflow incontinence

The notation of alkaline urine in a patient with a UTI may point to infection caused by: Klebsiella P. mirabilis E. coli S. saprophyticus

P. mirabilis

A common infective agent in domestic pet cat bites is: Viridans streptococcus species Pasteurella multocida Bacteroides species H influenzae

Pasteurella multocida

Which of the following is not characteristic of RA? It is more common in women than in men at 3:1 ratio Family h/o autoimmune conditions often is reported Peak age for disease onset in individuals is age 50-70 yrs Wrists, ankles, and toes often are involved

Peak age for disease onset in individuals is age 50-70 yrs

What is the treatment for Stress incontinence?

Pelvic floor rehab

Factors that contribute to stress incontinence include: Detrusor overactivity Pelvic floor weakness Urethral stricture UTI

Pelvic floor weakness

A 54yo white man w/no obvious risk for prostate cancer opted to undergo PSA screening and DRE testing. The DRE findings are normal and his PSA is 3.7. You recommend: Repeating the PSA test immediately Repeat screening in 1 year Repeat screening in 2 years Repeat screening in 5 years

Repeat screening in 1 year

Clinical presentation of type 1 DM usually includes all but: Report of recent unintended weight gain Ketosis Persistent thirst Polyphagia

Report of recent unintended weight gain

When making a home visit to an 89M who is bedridden, you note that he is cachectic, dehydrated, and cognitively intact. He states he is not receiving his meds regularly and that his granddaughter is supposed to take care of him but mentions "she seems more interested in my Social Security check." The patient is unhappy but asks that you tell nobody because he wants to remain in his home. The most appropriate action is to: Talk w/the granddaughter and eval her ability to care for the pt Visit the pt more frequently to ensure that his condition does not deteriorate Report the situation to the appropriate state agency Honor the pt's wishes because a competent pt has the right to determine care

Report the situation to the appropriate state agency

The Phalen test is described as: Reproduction of symptoms with forced flexion of the wrists Abnormal tingling when the median nerve is tapped Pain on internal rotation Palmar atrophy

Reproduction of symptoms with forced flexion of the wrists

Which of the following characteristics applies to type 1 DM? Significant hyperglycemia and ketoacidosis result from lack of insulin This condition is commonly diagnosed on routine exam or workup for other health problems Initial response to oral sulfonylureas is usually favorable Insulin resistance is a significant part of the disease

Significant hyperglycemia and ketoacidosis result from lack of insulin

A 52F has RA and p/w decreased tearing, "gritty"-feeling eyes, and a dry mouth. You consider a dx of: SLE Vasculitis Sjogren syndrome Scleroderma

Sjogren syndrome

Which of the following best describes the presentation of a person with RA? Worst symptoms in weight-bearing joints later in the day Symmetrical early morning stiffness Sausage-shaped digits with characteristic skin lesions Back pain with rest and anterior uveitis

Symmetrical early morning stiffness

Shingles most commonly involve the dermatomes of the: Legs and pubic area Face Upper arms and shoulders Thorax

Thorax

As part of an evaluation of a 3cm, round, mobile thyroid mass, you obtain a thyroid ultrasound scan revealing a fluid-filled structure. The most likely diagnosis is: Adenoma Thyroid cyst Multinodular goiter Vascular lesion

Thyroid cyst

In the report of a thyroid scan done on a 48F with a thyroid mass, a "cold spot" is reported. This finding is most consistent with: Autonomously functioning adenoma Graves' disease Hashimoto's disease Thyroid cyst

Thyroid cyst

A fixed, painless thyroid mass accompanied by hoarseness and dysphagia should raise the suspicion of: Autonomously functioning adenoma Graves' disease Hashimoto's disease Thyroid malignancy

Thyroid malignancy

Evidence-based factors that prevent or minimize the risk of UTIs include all of the following except: Male gender Longer urethra-to-anus length in women Timed voiding schedule Zinc-rich prostatic secretions

Timed voiding schedule

Which of the following is the least helpful test for the assessment of thyroid disease? Total T4 Thyroid-stimulating hormone (TSH) Free T4 Thyroid peroxidase (TPO) antibodies

Total T4

All of the following organisms have been implicated in the development of rosacea except: Viruses Bacteria Yeast Mites

Viruses

Osteoporosis prevention measures include all but: Calcium supplementation Selective estrogen receptor modulator use Vit B6 supplementation Weight-bearing and muscle-strengthening exercises

Vit B6 supplementation

Clinical findings of the knee in a pt with OA include all but: Coarse crepitus Joint effusion Warm joint Knee often locks or a pop is heard

Warm joint

Which of the following meds can contribute to the development of acute urinary retention in an older man with BPH? a. Amitriptyline b. Loratadine c. Enalapril d. Lorazepam

a. Amitriptyline

A 70-year-old patient comes to the clinic because she is worried about her memory. She worries that she is having more difficulty with remembering names and feels like her thinking is "cloudy and slower." She has been noticing these changes for the past three months. She is the caregiver for her mother, who has dementia and is concerned that she may also have the disease. She tells you that her mother is requiring much more care now and that it is getting harder to manage everything. She isn't able to visit friends or be part of her book club. She is tired and feels like "everything hurts." She is having trouble sleeping and feels anxious but doesn't feel depressed. Medical history is significant for HTN, GERD and cataracts. Medications are HCTZ and Pepcid. VS are normal. The patient is alert but anxious female with early cataracts bilaterally. Her cardiopulmonary exam is normal and her abdominal exam showed mild tenderness in the epigastric region. Her neuro exam is normal. Labs show a mildly decreased Vit B12 and an elevated methylmalonic acid level. What is you next step in caring for this patient? a. Administer the geriatric depression scale b. Consult psychiatry c. Order a CT of the head d. Prescribe cyanocobalamin e. Prescribe donepezil

a. Administer the geriatric depression scale

An 80-year-old patient comes to the clinic for a routine follow-up appointment. He has a 2-month history of increased fatigue and reports feeling as if he has "no energy at all." He had been the caregiver for his wife, who died one month ago. He is eating less and while he doesn't know how much weight he lost, he tells you that his pans are too loose now. He hasn't felt like going out for coffee with his usual bunch of friends and prefers staying in the house "watching TV." Through your ROS you find that he is having more knee and hip pain and is sleeping poorly. He is drinking two beers daily to help decrease his pain and while he manages this diabetes, he hasn't been checking his blood glucose lately. Medications are acetaminophen, lisinopril, atorvastatin, metoprolol and metformin. VS are normal. Physical exam shows an alert male, oriented to person, place and time. His mood is "here and there," with a dysphoric affect. His physical exam is WNL except for osteoarthritis findings in knees and hands, including crepitus in his knees. Cognitive screening using the MoCA tool shows a score of 27/30 and the geriatric depression scale screen reveals a score of 9/15. Lab results show normal CBC, COMP and TSH. His HbA1C is 7.5%. What is the next best step in caring for this patient? a. Ask the patient if they are having any thoughts of self-harm or suicide b. Order imaging c. Prescribe an antidepressant d. Refer the patient to psychology e. Screen for alcohol withdrawal

a. Ask the patient if they are having any thoughts of self-harm or suicide

A 78-year-old patient comes to the clinic with concerns of feeling tired and sad. She recently moved into an assisted living facility. She tells you she feels depressed and isn't interested in making new friends or joining any of the group activities. She states no suicidality. Her PMH includes HTN and diabetes. Her medications include lisinopril 5 mg daily, HCTZ 25 mg daily, and metformin 500 mg twice daily. Her physical exam and lab studies are unremarkable. The PHQ-9 score is 12. The patient is referred for psychotherapy and stared on sertraline for her depression. What is the best next step are her one-month f/u visit? a. Basic chemistry b. Blood glucose level c. CBC d. Echo e. Sertraline level

a. Basic chemistry

A 33 y/o F presents in April with fatigue, decreased energy level, and depressed mood for the last four weeks. After taking a thorough history you learn that this has coincided with a change in her sleep pattern and an increase in her food intake. She feels worthless both at home and at work and notes passive thoughts that "I might be better off dead." She has not had similar depressive symptoms in the past, but notes that she would have periods of time during her early 20s where she would be "very productive," often staying awake for several days at a time to complete projects. During that time, her friends reported that her behavior was atypical, as she would frequently do shopping and purchase large amounts of clothing. Currently, she does not have any symptoms od heat or cold intolerance, is sexually active in a monogamous relationship, does not have excessive anxiety, and is not having any hallucinations. What is the most likely diagnosis? a. Bipolar disorder b. Depression with psychotic features c. Generalized anxiety disorder d. Major depressive disorder e. Seasonal affective disorder

a. Bipolar disorder

A73 year old patient with a history of essential HTN and severe COPD on supplemental O2 2L NC presents to the clinic with new back pain. She reports that she was planting flowers in her garden when she tripped on the shovel and fell to the ground. Since then, she's had 7/10 lower back pain that is not improving with ice packs and acetaminophen. Her pain stays in her back and does not radiate down her legs. Medications also include salmeterol, albuterol, tiotropium and amlodipine. She is concerned that she cannot afford another health problem at this time as she's had 3 hospitalizations for COPD exacerbations in the last 3 month, taking a course of steroids and antibiotics with each hospitalization. She tries to stay active with zumba exercise class 3 times a week. VS are normal with O2 96% on 2L. BMI is 28. Physical exam shows a well appearing female in mild discomfort. She is breathing comfortably with decreased breath sounds over bilateral posterior lung fields and no audible wheezing. Cardiac and abdominal exams are normal. There is tenderness to palpation in the lower lumbar vertebrae without crepitus. Her gait is low with normal strides. Sensation intact with soft touch over bilateral feet. An XR of her lumbar sacral spine confirms an acute L3-L4 vertebral fracture. Which aspect of this patient's history is most likely to be associated with diagnosis of osteoporosis? a. Exposure to steroids b. Her age c. Her BMI d. Low impact exercise e. Prescription amlodipine

a. Exposure to steroids

When assessing a 78yo w/suspected BPH, the NP considers that: a. Prostate size does not correlate well w/severity of symptoms b. BPH affects less than 50% of men his age c. He is at an increased risk for prostate cancer d. Limiting fluids is a helpful method of relieving severe symptoms

a. Prostate size does not correlate well w/severity of symptoms

You are counseling a female with a new diagnosis of major depression regarding her treatment options. After discussing the common side effects, she is willing to start medication. However, she seems reluctant to seek counseling services. You help to diffuse her perception that counseling is only for those who are "weak," but she remains reluctant. Which of the following statements should you tell her regarding psychotherapy for treatment of mild-to-moderate depression? a. Psychotherapy can be as effective as medication for treatment of mild-to-moderate depression b. Psychotherapy is effective when combined with pharmacotherapy, but not effective when used alone to treat depression c. Psychotherapy is most commonly conducted by a psychiatrist, rather than any other mental health professional d. Psychotherapy is not effective for the treatment of mild-to-moderate depression e. Psychotherapy is reserved for patients with suicidal thoughts

a. Psychotherapy can be as effective as medication for treatment of mild-to-moderate depression

A 75-year-old patient is seen in the clinic with a concern for a depressed mood. He has had depression in the past, which was successfully treated with paroxetine. He now reports sadness, lack of energy, feelings of helplessness and apathy. Past medical history is significant for depression (age 45), CAD, HTN and CKD stage 2. Medications include ASA 81 mg daily, metoprolol 25 mg twice daily, lisinopril 10 mg daily and simvastatin 20 mg daily. His VS are WNL and his physical exam is unremarkable. What medications would you recommend to treat this patient's depression? a. Sertraline b. Lorazepam c. Methylphenidate d. Mirtazapine e. Paroxetine

a. Sertraline

According to recent epidemiologic studies, prostate cancer is the number ____ cause of death in U.S. men. a- 1 b- 2 c- 3 d- 4

b- 2

When prescribing antiHTN therapy for a man w/BPH and HTN, the NP considers that: a. Loop diuretics are the treatment of choice b. An alpha1 antagonist should not be used as a solo or 1st line agent c. Angiotensin receptor antagonist use is contraindicated d. Beta-adrenergic antagonist use often enhances urinary flow

b. An alpha1 antagonist should not be used as a solo or 1st line agent

A 75-year-old patient is seen via video clinic appointment. Her main concern is fatigue. She has been isolating herself from family and friends in efforts to protect herself against a virus which has been spreading through her community. She is tearful and admits to "feeling down." She used to enjoy her book club and activities at church but tells you she just "has no interest in getting out anymore." Medical history is significant for cataracts, lumbar spinal stenosis with sciatica to bilateral legs, osteoporosis and falls. Medications are acetaminophen 1000 mg twice daily, calcium supplement 200 mg twice daily, vit D 1000 IU daily and alendronate 70 mg weekly. You are unable to check her VS. The patient appears tired and sad. She looks away from the video monitor during the visit and apologizes to you for her "messy room with all this clutter." Physical exam shows downcast eyes and head, some redness to her sclera bilaterally. Respirations appear comfortable and unlabored. What is your next step in caring for this patient? a. Begin treatment with a low dose SSRI b. Order a thyroid stimulating hormone level c. Order an echo d. Refer for counseling e. Schedule a f/u appt in 4 weeks

b. Order a thyroid stimulating hormone level

The average American man has an approximately ____ % lifetime risk of prostate cancer and an approximately ____ & likelihood of clinical dz? a. 15, 5 b. 25, 8 c. 40, 10 d. 60, 15

c. 40, 10

A 72 y/o patient is brought to the ED for bilateral leg pain. He reports at least 2 years of low back pain but over the last 2 months, he notes pain in bilateral posterior thighs and calves. He reports no inciting event. He describes the pain as burning, cramping and numbness whenever walking. Over the last week, he feels he cannot even walk a block without needing to stop. He also noted the pain when standing during his church choir practice has started leaning over the pew in front of him. He has no pain when sitting or lying down at night when sleeping. He reports no change in bowel or bladder habits. VS are normal. Physical exam shows a patient with no distress when seated, but during the gait exam the patient is bent forward and grimaces. There is no tenderness to palpation over the spinal processes. Rotation and flexion of back is WNL, but the patient is guarded with extension of the back. Reflexes at patella and Achilles are normal bilaterally. Strength and sensation are intact. Straight leg raise negative bilaterally. Normal posterior tibial and dorsalis pedis pulses bilaterally. Results of lab studies are unremarkable. What test is mostly likely to confirm the diagnosis? a. Ankle brachial index b. Electromyography c. MR of the lumbar spine d. PET scan e. XR lumbar spine

c. MR of the lumbar spine

A 32 y/o post-partum female presents with splotchy dark spots that are otherwise asymptomatic. She reports no acne or other skin conditions such as rosacea. On exam, you note hyperpigmented macules on the upper lip. Which of the following is the most likely diagnosis? a. Cafe au lait spots b. Fixed-drug eruption c. Melasma d. Post inflammatory hyperpigmentation e. Solar lentigo

c. Melasma

Concerning herbal and nutritional therapies for BPH tx, which of the following statements is false? a. The mechanism of action of the most effective and best studied products is similar to prescription meds for this condition b. These therapies are currently considered emerging therapies by the American Urological Association c. Major areas of concern with use of these therapies include issues of product purity and quality control d. These therapies are safest and most effective when used with prescription meds

d. These therapies are safest and most effective when used with prescription meds

An 80 y/o patient comes to the clinic because of a 4-day history of low back pain radiating to the buttocks. The pain began while he was lifting boxes in his garage, and increased 25 hours after it initially began, since then it has been unremitting and refractory to acetaminophen 500mg twice daily and ibuprofen 400 mg twice daily. Medical history is significant for HTN, hypercholesterolemia, CKD, non-insulin dependent DM, prostate cancer treated with external beam radiation, osteoarthritis and GERD. Medications include atenolol, metformin, simvastatin, tamsulosin and omeprazole. Vital signs are normal. Physical exam is remarkable for point tenderness in his upper lumbar vertebra. He has pain with straight leg raise bilaterally starting at 45 degrees and relieved with knee flexion. He has no pain with internal or external rotation of his hips. Pain is increased with rotation of his torso, sitting up or initially when lying down. There is no pain with arm movement on either side. There are no focal or neurologic deficits or numbness. Gait is slow. Based on his history and exam, what is your next step in the evaluation of this patient? a. Complete blood count b. Erythrocyte sedimentation rate c. Nuclear bone scan d. Plain XRs of the lumbar spine e. Prostate specific antigen

d. Plain XRs of the lumbar spine

A 78yo presents with a 3d h/o new-onset fatigue and difficulty with bladder emptying. Exam reveals a distended bladder but is otherwise unremarkable. BUN is 88, creat 2.8. The most likely dx is: a. Prerenal azotemia b. Acute glomerulonephritis c. Tubular necrosis d. Postrenal azotemia

d. Postrenal azotemia

All of the following can cause an elevated PSA level except: a. Current prostate infection b. Recent cystoscopy c. BPH d. Prostatectomy

d. Prostatectomy

A 69-year-old patient comes to the clinic because of a 4-day history of mid low back pain. He does not recall any recent trauma, but states he helped his daughter move last week where he lifted many large boxes. He reports that there is no radiation or electric sensation with his pain. The pain improves with lying flat and is worse with twisting. Pain is relieved with 1-2 tabs of regular strength acetaminophen. He reports losing 20 pounds in the last 4 months and has not changed his diet or daily routine. Medical history is significant for HTN, and hyperlipidemia. Medications are atorvastatin and lisinopril. He stopped smoking over 10 years ago and has an 18-pack year history. He drinks 1-2 beers a week. Vital signs are normal. Physical exam shows a gentleman in mild discomfort. Cardiopulmonary exam is unremarkable. There is pain to palpation of his mid lower back and with active ROM. Sensation and strength in the lower extremities is intact. Results of lab studies show hemoglobin of 10.8 and creatinine .92. Which elements of this patient's history is consistent with a concerning non-mechanical cause of lower back pain? Choose the single best answer. a. Improvement in pain upon lying flat b. Lack of radicular component to pain c. Short duration of pain d. Weight loss in the last 4 months e. Worsening pain with twisting motion

d. Weight loss in the last 4 months

What is the treatment for Transient incontinence?

treating an underlying cause


Set pelajaran terkait

Telecom Chapter 5 - Ethernet (802.3) Switched LANs

View Set

Biology - Deforestation, Greenhouse effect and Greenhouse gases.

View Set